Cardiology Flashcards Preview

Fess' Flashcards: 6th Year > Cardiology > Flashcards

Flashcards in Cardiology Deck (57)
Loading flashcards...
1
Q

What are the ECG criteria for a STEMI?

A

Elevation in at least two contiguous leads of 2mm in the precordial leads, and 1mm in the limb leads

2
Q

Give 3 indications to attempt rhythm control rather than rate control for AF

A

New onset (within 48 hours)
Reversible cause
The atrial fibrillation is causing heart failure
Young age/ few comorbidities
Rate control is difficult to achieve/ symptomatic even after rate control

Link to ESC flowchart on rhythm control:
https://oup.silverchair-cdn.com/oup/backfile/Content_public/Journal/eurheartj/PAP/10.1093_eurheartj_ehaa612/3/ehaa612f15.png?Expires=1609322446&Signature=0KcCkSeRXpk4JtA25clSB-xp2gbqukTr4D~pGAPUiHbLWyrsFDmgqx93FV-4jj43yDuw4L4lDK7Q3iqL4wV6s2ElMPH0OsroWkflzo7BGOv~5N74EDdsUykwTbFcb60Tr-j-H8pKmmmUfyOnCvigsRZzZW~fAynC~PTB-CypDPOeDnpEu0Mg4aheokjlPuon78GPSVCo1WE5jGdzKukXOTDJQWD6jY820KzidvUsXgGGfdqGViywuWkWIAvtiYgt3KZ0r2jO5LndCO4HOYrh~GkvAF0Ua~6EL0Fg5XhN2gW1QMlYsv-xWzie6l-gnJGHSt5~lkCqLBvwkUEVin3PSA__&Key-Pair-Id=APKAIE5G5CRDK6RD3PGA

3
Q

The F2 on call is bleeped to review a 67 year old woman who has developed breathlessness. She was admitted 2 days ago with a STEMI and is now SOB, coughing up frothy sputum, and saturating at 92% on room air. The nurse sits her up and gives 15L 100% oxygen through a non-rebreathable mask. On examination her peripheries are warm, and auscultation reveals bilateral fine crackles and expiratory wheeze. Her BP is 123/85, HR is 93. Her ECG features t wave inversion and pathological q waves.

How should this patient be managed?

A. Put out a 2222 call
B. Give I.V. inotropes and vasopressors
C. Contact the critical care outreach team with a view to haemofiltration and cardiac support
D. Defibrillation
E. Give I.V. furosemide and nitrates
A

E. Give I.V. furosemide and nitrates

This is a history of acute heart failure which can be classified by whether it is ‘wet’ or ‘dry’ (congestive vs. non-congestive) and whether it is ‘warm’ or ‘cold’ (is peripheral perfusion intact?): this will determine the management. 95% of cases are ‘wet’ and feature congestive features such as pulmonary oedema and peripheral oedema. Diuretics and vasodilators (e.g. nitrates) are commonly used for acute heart failure, but should be avoided if there is poor peripheral perfusion as they will worsen it. Vasopressors and inotropes may well be needed but will be given in an ICU setting, so although contacting critical care outreach is a good idea, the first priority in this woman (who is warm and well-perfused) is to give nitrates and furosemide.

NB: though furosemide’s properties as a diuretic are helpful here, its purpose in acute heart failure is actually venodilation to reduce preload on the heart

NB: the NICE guidelines aren’t hugely helpful for this topic, you are better off looking at the European guidelines and understanding the principles behind treatment

ESC flowchart for heart failure:
https://academic.oup.com/eurheartj/article/37/27/2129/1748921#109987142

ESC guidelines on heart failure:
https://oup.silverchair-cdn.com/oup/backfile/Content_public/Journal/eurheartj/37/27/10.1093_eurheartj_ehw128/4/ehw12805.jpeg?Expires=1617535354&Signature=lu6fF~DmdyGgFz8cN1VVA~wj2a6ycvaZGkoh1Krr1WTs6oevA8loxNEI9kDToeqivvBwJXju08nCQx8H4xO64kArH1z5fdZZ9hD1oH-KAymJTAw1s28NAVhD1QZWajGu8Wq5CO7pW6HT-~XBdlRwREXRpNvCCmOwusXk3y8RM39To6uXd9ZGjvxdkAR3T3naHLx3cCUS1ou2Vo0t1LajVWb~023CVw3J71aorMyyqKpzk142N32CuiEq7Zm2XRYnf8wsBCg9Qo4NHuyJtVU9S-9WEekvLXzFIATjTjVG9XwP2FyuoTjcrutdQEHu3Ogt~4ACxHYJ5Tpv00RqBZhQgw__&Key-Pair-Id=APKAIE5G5CRDK6RD3PGA

4
Q

What is the management of chronic heart failure with reduced ejection fraction?

A

The aim is to get the ejection fraction above 35%

Furosemide is used for symptomatic relief
1st Start ACEi and beta blockers
If EF still <35%, add mineralocorticoid antagonist
If EF still <35%, consider an angiotensin receptor/neprilysin inhibitor combination (ARNI) e.g. sacubitril/valsartan, CRT, or Ivabradine
If EF still <35%, consider digoxin, LVAD, transplant

NB: CRT involves pacemaker leads in the right atrium, right ventricle, and a cardiac vein off the coronary sinus (over left ventricle). The patient is then blocked with beta blockers, and the pacemaker will take over the majority of the work initiating contractions.

ESC flowchart:
https://oup.silverchair-cdn.com/oup/backfile/Content_public/Journal/eurheartj/37/27/10.1093_eurheartj_ehw128/4/ehw12802.jpeg?Expires=1614428667&Signature=B06hufiT~ZHtVEAJI9iAHpw0yH6Kr8s2X9e895zX3DYBrYfHlg4BJNKxZtNWCvHNSDm6baxmFtylSp7V3NHqrzRch908mx4NADvIoDFdjI2CTnT61RCgHXK4pR~yOfhxaZxTyuKru-zWDeQ16hBuoCQgPZDPSflgrOwm6BzQOb8lfmHsU5Fs3RZRvie9KyT0p8-4PCbeC6ngJ8RF-iW9u~DLgjOg4D930KGUcq0qB7jfzBeGgv9Fvq13SkGe2sFT-xM5fgvy8NgNRVIhzo5XH3w0fGl8sq8aqFJqGzr-TPv94mjqirIHzUr~SCmUS73RqI0L9FQHxzazzd4Al3x2yQ__&Key-Pair-Id=APKAIE5G5CRDK6RD3PGA

5
Q

Which medications should be given to a patient diagnosed with a STEMI immediately before they are sent for PCI?

A

Aspirin 300mg
Preferably Prasugrel 60mg (alternatively Ticagrelor 180mg or Clopidogrel 300mg)
Oxygen (if sats <90%)
Morphine (if required and not hypotensive)
Unfractionated heparin (radial access) or bivalirudin (femoral access) is offered in the cathlab

6
Q

A 66 y/o man presents with a history of central constricting chest pain that is brought on by exertion and relieved by rest. He is diagnosed with stable angina. He has high cholesterol and asthma, and is taking an LRTA regularly.

Which medication should he be started on to prevent episodes from occurring?

A. Nicorandil
B. Bisoprolol
C. Atenolol
D. Diltiazem
E. GTN spray
A

D. Diltiazem

First line treatment for prevention of episodes of stable angina is either a CCB or a beta blocker, however this man’s asthma is a contraindication to beta blockers. GTN spray is given to these patients, but is a reliever therapy, not a preventative. Patients whose angina is not controlled with one option should then be tried either on the other option or on both. In this case, if diltiazem were insufficient in controlling angina one of the following may be added:

A long-acting nitrate
Ivabradine
Nicorandil
Ranolazine

7
Q

Describe the treatment algorithm for hypertension

A

Hypertension is usually treated with pharmacological intervention at stage 2 or above (>160/100), but may be treated at stage 1 (140/90 - 160/100) if there are comorbidities (diabetes, renal disease, end organ damage, previous cardiovascular disease, or a QRISK of 10% or more)
NB: treatment should be started in these patients, but this does not mean patients with stage 1 hypertension and without these comorbidities should not be treated - but the decision is more individual and less clear cut.

Step 1:
Start CCB in patients if they are over 55 or black
Start ACEi or ARB in all other patients including any with T2DM (including >55 years old or black)

Step 2:
Add 1 drug from the other category (e.g. add a CCB to an ACEi, or an ARB to a CCB) or add a thiazide-like diuretic (e.g. indapamide)

Step 3:
Give a CCB, a thiazide-like diuretic, and an ACEi/ARB

Step 4:
Add spironolactone (should only be given if K+ is below 4.5mmol/L) or an alpha/ beta blocker
8
Q

Which drug should be given as first line for rate-control in atrial fibrillation?

A

Either a typical beta blocker (i.e. not Sotalol) or a rate limiting CCB
Digoxin monotherapy may be considered for very sedentary patients
If monotherapy fails, consider a combination of 2 of:

A beta blocker
Diltiazem
Digoxin

9
Q

A 67 year old woman visits her GP for a check-up following her ambulatory blood pressure measurement, which has confirmed she has hypertension. She has previously been diagnosed with hypercholesterolaemia and type II diabetes, and the doctor gives her advice on lifestyle measures she can implement.

What is the next step in her management?

A. Record her BP again in 3 months
B. Begin a CCB
C. Begin an ACEi
D. Begin a beta blocker
E. Record her BP again in 6 months
A

C. Begin an ACEi

Hypertension is diagnosed by the presence of an in-clinic reading of ≥140/90 AND an ambulatory or home BP average of ≥135/85. Since this patient is under 80 and has diabetes, pharmacological treatment is indicated rather than lifestyle modification alone. Though a patient over 55 would normally be started on a CCB or thiazide-like diuretic, the diagnosis of type II diabetes means she should be started on an ACEi or ARB.

10
Q

Which of the following medications is contraindicated in combination with beta blockers?

A. Amiodarone
B. Amlodipine
C. Verapamil
D. GTN
E. Ramipril
A

C. Verapamil

Verapamil and other rate-limiting CCBs (e.g. Diltiazem) produce complete heart block when used in conjunction with beta blockers.

11
Q

The F1 on call is bleeped to see a patient who began to have severe central curshing chest pain 10 minutes ago. The pain radiates to their right arm and is constant, and is associated with sweating and nausea. The nurse hands the F1 an ECG which shows a normal P wave and QRS complex, but tall, broad, asymmetrical T waves in the anterior leads.

What is the most likely diagnosis?

A. Pulmonary embolism
B. STEMI
C. Hyperkalaemia
D. NSTEMI
E. Unstable angina
A

B. STEMI

The tall, broad, asymmetrical t waves described here are hyperacute t waves. This is a description of the very early stages of a STEMI, where hyperacute t waves may be the first change. If this ECG were repeated a short while later, it would show ST elevation in the anterior leads.

12
Q

How is idiopathic pericarditis with no complications or large effusion treated?

A

Colchicine and Aspirin/ NSAIDs with gastroprotection (e.g. PPI)

13
Q

What is the most common cause of death following a myocardial infarction?

A

Ventricular fibrillation

14
Q

Recall which valvular defect would correspond to each auscultation finding below:

A. An early-diastolic murmur best heard in left parasternal region
B. An opening snap and mid diastolic rumbling murmur heard in the 5th intercostal space in the midclavicular line
C. Pan systolic heard louder on inspiration
D. Ejection systolic murmur that does not radiate

A

A. An early-diastolic murmur best heard in left parasternal region - aortic regurgitation
B. An opening snap and mid diastolic rumbling murmur heard in the 5th intercostal space in the midclavicular line - mitral stenosis
C. Pan systolic heard louder on inspiration - tricuspid regurgitation
D. Ejection systolic murmur that does not radiate - HOCM or aortic sclerosis

15
Q

What do each of the following heart sounds represent?

S1
S2
S3
S4

A

S1: Closure of mitral and tricuspid valves
soft if long PR or mitral regurgitation
loud in mitral stenosis

S2: Closure of aortic and pulmonary valves
soft in aortic stenosis
splitting during inspiration is normal

S3 (third heart sound): Caused by diastolic filling of the ventricle
Considered normal if < 30 years old (may persist in women up to 50 years old)
Heard in left ventricular failure (e.g. dilated cardiomyopathy), constrictive pericarditis (called a pericardial knock) and mitral regurgitation

S4 (fourth heart sound): Caused by atrial contraction against a stiff ventricle, and therefore coincides with the P wave on ECG
May be heard in aortic stenosis, HOCM, hypertension
In HOCM a double apical impulse may be felt as a result of a palpable S4

16
Q

A 58 year old man presents to A&E with breathlessness. The problem has been bothering him for several weeks and has not resolved spontaneously. His pulse is irregularly irregular, and an ECG shows a narrow complex irregular rhythm with a lack of P waves. He is sent for an echocardiogram which shows a dilated left atrium. He has a background of an MI 5 years ago that has left him with left bundle branch block, and type 2 diabetes.

Which medication should this man be started on?

A. Flecainide
B. Amiodarone
C. Amlodopine
D. Bisoprolol
E. Adenosine
A

D. Bisoprolol

This man is presenting with several weeks worth of palpitations, and so the first step is to rate control him and anticoagulate him for at least 3 weeks.

Given the atrial dilatation and ischaemic heart damage, this man is unlikely to be a good candidate for rhythm control (cardioversion or an interventional procedure). Instead the goal should be rate control, for which the first line drug is either a beta blocker or a rate limiting CCB.

This man will also need anticoagulating long-term as his CHADSVASc score is 2.

17
Q

Calculate each of these patients’ CHA2DS2-VASc scores:

A. A 79 year old man with HTN and hypercholesterolaemia, who had an MI 5 years ago

B. A 45 year old woman with T2DM and a DVT 2 years ago

C. A 68 year old woman with a history of stroke and congestive cardiac failure

A

A. A 79 year old man with HTN and hypercholesterolaemia, who had an MI 5 years ago - 4

B. A 45 year old woman with T2DM and a DVT 2 years ago - 4

C. A 68 year old woman with a history of stroke and congestive cardiac failure - 5

The scoring system is as follows:
C - congestive heart failure (or left ventricle dysfunction)
H - hypertension
A2 - age 75+
D - diabetes mellitus
S2 - previous stroke/TIA/VTE
V - vascular disease (e.g. peripheral arterial disease, MI, angina)
A - age 65-74
Sc - sex category (+1 for female, nothing for male)

NB: Hypertension is counted whether it is controlled or not
NB: Hypercholesterolaemia does not score a point on its own
NB: the threshold for considering anticoagulation is 1 or more in men, and 2 or more in women.

18
Q

Which artery usually supplies the posterior descending artery, and how common is this variant?

A

The PDA is usually supplied by the right coronary artery (80-85% of people) with other people being either left coronary artery dominant, or co-dominant.

19
Q

What are the two ways to diagnose a posterior MI on ECG?

A

1) Put leads on the patient’s back
2) V1-V3’s QRS complexes will become overall positive because of the loss of negative signal from the depolarisation of the posterior segment of the heart. Classically there is a positive R wave in V2

20
Q

Which of the following drugs does NOT improve long-term prognosis of heart failure?

A. Furosemide
B. Bisoprolol
C. Ramipril
D. Spironalactone

A

A. Furosemide

Though very useful in treating acute heart failure given its properties as a loop diuretic and venodilator, the evidence does not show a long term reduction in mortality with furosemide therapy. However this does not mean it is not useful and important to give in fluid overloaded patients. All the other options have been proven to improve long term prognosis.

21
Q

A 72 year old man is brought by ambulance to A&E with a 1 hour history of central crushing pain radiating to the left shoulder and jaw. His ECG in the ambulance shows ST elevation in leads II, III, and aVF. Whilst being clerked by the FY2, the man suddenly develops breathlessness. His BP is 75/40 (down from 110/78 on admission) and his SO2 is 91% (down from 97% on admission). Auscultation reveals bibasal lung crepitations.

What is the most likely cause of the sudden deterioration?

A. Ventricular aneurysm
B. Ventricular rupture
C. Pulmonary embolism
D. Haemorrhage
E. Mitral valve papillary rupture
A

E. Mitral valve papillary rupture

Mitral valve papillary muscle rupture is a complication of myocardial infarction particularly with infero-posterior infarction. It causes acute mitral regurgitation which leads to flash pulmonary oedema.

22
Q

What is the most common cause of mitral stenosis worldwide?

A

Rheumatic fever

23
Q

When do troponin levels generally peak post-ACS?

A

18-24 hours

24
Q

Give 5 causes of atrial fibrillation

A
Sepsis
Ischaemic heart disease
Electrolyte abnormalities
Drugs
Structural heart disease e.g. HOCM
Hyperthyroidism
25
Q

A 55 year old woman is brought to hospital by ambulance with sudden onset crushing chest pain radiating to her jaw and right arm. The paramedics performed an ECG before arriving which shows significant ST depression in leads V5, V6, and I. She is given 300mg aspirin as well as sublingual GTN. She is assessed by cardiology when she arrives at hospital and is unstable so they decide immediate PCI is appropriate.

Which additional medications should this patient be given?

A. Clopidogrel and fondaparinux
B. Prasugrel and enoxaparin
C. Prasugrel and unfractionated heparin
D. Ticagrelor and enoxaparin
E. Clopidogrel and unfractionated heparin
A

C. Prasugrel and unfractionated heparin

In this scenario where there is an NSTEMI and immediate PCI is planned, fondaparinux should not be given. However planned PCI within 24 hours is no longer a contraindication to fondaparinux; previously there was evidence that fondaparinux increased the risk of catheter thrombosis in PCI, but it has now been shown that addition of unfractionated heparin removes this risk.

Prasugrel is a potent anti-platelet agent that should be added to aspirin in patients who are scheduled to undergo PCI. In patients with a higher bleeding risk, use clopidogrel instead.

Unfractionated heparin is given to prevent catheter thrombosis.

Flowchart summary of NSTEMI guidelines:
https://www.nice.org.uk/guidance/ng185/resources/visual-summary-unstable-angina-nstemi-pdf-8900622109

NICE ACS guidelines:
https://www.nice.org.uk/guidance/ng185/chapter/Recommendations

26
Q

A 55 year old woman is brought to hospital by ambulance with sudden onset crushing chest pain radiating to her jaw and right arm. The paramedics performed an ECG before arriving which shows significant ST depression in leads V5, V6, and I. She is given 300mg aspirin as well as sublingual GTN. She is assessed by cardiology who diagnose NSTEMI: she is low-risk in terms of future mortality (GRACE <3%) and they are happy for her to be managed medically.

Which additional medications should this patient be given?

A. Clopidogrel and fondaparinux
B. Prasugrel and enoxaparin
C. Prasugrel and unfractionated heparin
D. Ticagrelor and enoxaparin
E. Ticagrelor and fondaparinux
A

E. Ticagrelor and fondaparinux

Flowchart summary of NSTEMI guidelines:
https://www.nice.org.uk/guidance/ng185/resources/visual-summary-unstable-angina-nstemi-pdf-8900622109

NICE ACS guidelines:
https://www.nice.org.uk/guidance/ng185/chapter/Recommendations

27
Q

Which medications should patients be discharged on post-ACS?

A
Aspirin (lifelong)
Clopidogrel (12 months)
ACE inhibitor
Beta blocker
Statin
\+ Spironolactone if reduced LVEF
28
Q

Summarise the management of angina:

A

1st line beta blocker and/or CCB for episode prevention
If one isn’t tolerated switch
If one is inadequate switch or combine
If both are unsuitable consider monotherapy with one of:
Long-acting nitrate (such as isosorbide mononitrate)
Nicorandil
Ivabradine
Ranolazine
If optimal medical treatment does not control treatment, ocnsider evascularising with PCI or CABG

GTN spray for symptom relief
Give a statin
Give 75mg aspirin for prevention
Start an ACEi if they are diabetic
Treat hypertension if present
Lifestyle modification
29
Q

A 62 year old woman visits her GP complaining of fatigue and breathlessness. She was taken to hospital 4 weeks ago for a STEMI, but was managed medically due to her delayed presentation. On exmaination her JVP is 2cm and there is no peripheral oedema. On auscultation there are scattered wheezes throughout her chest and bibasal crackles. An ECG taken by the GP reveals the same ST elevation seen 4 weeks ago.

Which complication of myocardial infarction has ocurred?

A. Papillary muscle rupture
B. Ventricular aneurysm
C. Dressler syndrome
D. Congestive heart failure
E. Ventricular septal rupture
A

B. Ventricular aneurysm

Persistent ST elevation is a giveaway for ventricular aneurysm, which also causes left heart failure (pulmonary oedema causing wheeze and crackles). Ventricular aneurysms are weakenings in the left ventricle wall where tissue has necrosed and formed bulging pockets. These pockets allow thrombi to form and may lead to thromboembolic events such as stroke or mesenteric ischaemia.

30
Q

JVP waveform matching question:

  1. Cannon a waves
  2. Giant v waves
  3. Kussmaul’s sign
  4. Absent a waves

A. Atrial fibrillation
B. Tricuspid regurgitation
C. Complete heart block
D. Constrictive pericarditis

A
  1. Cannon a waves - C. Complete heart block
  2. Giant v waves - B. Tricuspid regurgitation
  3. Kussmaul’s sign - D. Constrictive pericarditis
  4. Absent a waves - A. Atrial fibrillation

JVP pressure waveforms consist of ‘a’, ‘c’, ‘x’, ‘v’, and ‘y’ segments.

‘A’ is a pressure increase resulting from right atrial contraction. This will be absent/ reduced in atrial fibrillation because there is a lack of coordinated atrial contraction. The ‘a’ waves will be more pronounced at times in 3rd degree heart block because of the dissociation between atrial and ventricular contraction: the right atrium will at times attempt to contract against a contracted ventricle which will increase the pulse pressure during the ‘a’ wave - cannon ‘a’ waves.

‘C’ corresponds to closure of the tricuspid valve which causes a small pulse of back pressure into the jugular. Ths is not usually visible on the waveform but if vivisble will cause a slight upwards deviation in the downwards slope of the ‘a’ wave.

‘X’ is the descent after the ‘a’ wave as the ventricles contract and the right atrium relaxes.

‘V’ is the ascent after ‘x’ as the right atrium passively fills against a closed tricuspid valve. ‘X’ waves are more pronounced/ giant in tricuspid regurgitation because the right ventricle pushes blood back into the atrium, increasing the pressure.

Finally, the ‘y’ descent is the opening of the tricuspid valve.

Kussmaul’s sign is a rise or lack of fall in the JVP on inspiration, and is caused by impaired right atrial filling e.g. due to pericardial effusion, constrictive pericarditis, or severe right heart failure.

31
Q

Which of the following ECG abnormalities would NOT typically be seen in hyperkalaemia?

A. Broad QRS complexes
B. Inverted T waves
C. Prolonged PR interval
D. Sinusoidal rhythm
E. Absent P waves
A

B. Inverted T waves

Hyperkalaemia classically produces ‘tall tented t-waves’. Hyperkalaemia causes a progression of ECG changes:
FIrst the T waves tent
Then the P-R interval lengthens
Then the P-waves flatten
Then the QRS prolongs
Then a sinusoidal rhythm develops leading to arrest

https://litfl.com/hyperkalaemia-ecg-library/

32
Q

A 56 year old man presents to A&E with 1 hour of crushing central chest pain that radiates to his neck and is 9/10 in severity. The chest pain came on at rest. He is sweating and looks generally ill. A rapid ECG reveals ST depression in leads V1-V3, with a dominant R wave in V2.

What is the most likely diagnosis?

A. Unstable angina
B. Anterior STEMI
C. Posterior MI
D. NSTEMI
E. Stable angina
A

C. Posterior MI

The posterior heart is supplied by the posterior descending artery ( a branch of the right coronary artery in 80-85% of people). Infarction of this territory is difficult to spot because ECG leads are not usually placed on the back, so the signs are more subtle than a classic ST elevation picture.

ST depression in the anterior leads will occur as it is a mirroring of the ST elevation that would be seen on posterior leads. The other main change is positive R waves in V1-V3. The R waves in these leads are usually negative because the depolarisation of the posterior heart is also captured in these leads which makes the complexes overall negative. When the posterior heart is infarcted, this overlaid effect is lost and the complexes of these leads will become positive.

33
Q

A 54 year old man presents to A&E with an acute onset 1 hour episode of ongoing palpitations and mild dyspnoea. An ECG shows a sawtooth baseline where not every p wave is followed by a QRS complex, though the relationship between them seems random. The rate is irregularly irregular at ~150bpm.

What is the most likely diagnosis?

A. Fast atrial fibrillation
B. Supraventricular tachycardia
C. Atrial flutter with variable block
D. Atrial flutter with 2:1 block
E. Atrio-ventricular reentrant tachycardia
A

C. Atrial flutter with variable block

Atrial flutter is characterised by a rhythm that circles a reentrant circuit in the atria at a very high rate (~300bpm): this causes a ‘sawtooth baseline’ appearance. Not all of these rhythms are conducted to the ventricles, and ithe relationship may be fixed or variable. This presentation may be tricky to differentiate from fast atrial fibrillation, but here there is a clear description of p-waves which eliminates atrial fibrillation as a differential.

34
Q

Which of the following describes true tri-fasicular block?

A. 1st degree heart block, RBBB, and left axis deviation
B. 2nd degree heart block, LBBB, and right axis deviation
C. 1st degree heart block, LBBB, and right axis deviation
D. 3rd degree heart block, RBBB, and left axis deviation
E. 3rd degree heart block, LBBB, and right axis deviation

A

REVIEW*

D. 3rd degree AV block, RBBB, and left axis deviation

NB: can be either left or right axis deviation depending on whether the left anterior or posterior fascicle is affected

35
Q

What is the time frame within which PCI is automatically given for a STEMI?

A

Within 12 hours of onset of symptoms, and if PCI can be given within 2 hours of presentation

36
Q

Briefly describe the New York Heart Association’s classification for heart failure:

A

Class I - No symptoms and no limitation in ordinary physical activity, e.g. shortness of breath when walking, climbing stairs etc.
Class II - Mild symptoms (mild shortness of breath and/or angina) and slight limitation during ordinary activity.
Class III - Marked limitation in activity due to symptoms, even during less-than-ordinary activity, e.g. walking short distances (20-100 m).Comfortable only at rest.
Class IV - Severe limitations. Experiences symptoms even while at rest. Mostly bedbound patients

37
Q

A STEMI in which territory is most likely to cause heart block?

A. Septal
B. Anterior
C. Lateral
D. Inferior
E. Posterior
A

D. Inferior

The inferior territory is supplied by the right coronary artery, which also supplies the AV node. Infarction of the blood supply to the AV node leads to conduction delay, heart block, and arrhythmias.

38
Q

A 36 year old woman presents to A&E with a 6 day history of central chest pain. She also reports 2 days of breathlessness that is worse when lying down, fatigue, and palpitations. She has previously been fit and well with no cardiac issues. She gave birth to her 2nd child 3 weeks ago: the birth was uncomplicated and she has been using barrier contraception since. She mentions that she had a bout of flu-like symptoms 2 weeks before.

On examination she has bilateral basal fine crackles, HS 1+2+0, abdomen SNT, calves SNT. Her initial investigations are as follows:

HR - 112bpm
BP - 115/79mmHg
RR - 26bpm
SO2 - 93% RA
ECG - diffuse t-wave inversion
Troponin - significantly elevated
CRP - 35mg/L
CXR - shows bilateral lower zone fluffy opacities

What is the most likely diagnosis?

A. Takotsubo cardiomyopathy
B. Pulmonary embolism
C. NSTEMI
D. Dilated cardiomyopathy
E. Myocarditis
A

E. Myocarditis

Myocarditis is defined as inflammation of the myocardium with a non-ischaemic cause. It is essentially a sign of other diseases, and so has a range of causes:

Infection:
Worldwide the most common cause is Chagas disease, with other common agents being adenoviruses, enteroviruses, Parvovirus B19, and even Hepatitis C.

Autoimmune:
It is associated with multiple autoimmune disorders including scleroderma, SLE, and sarcoidosis.

Toxic:
A variety of drugs, venoms, and heavy metals may cause myocarditis but this is less common.

In this case the patient has presented with features of congestive heart failure due to their myocarditis: dyspnoea and orthopnoea in addition to the other myocarditis symptoms of pain, palpitations and fatigue. The key risk factors in this question are the preceding viral infection and the fact she is post-partum. Her young age is also an indicator as ACS in a woman this age would be unusual. Though not present here, HIV infection is a very strong risk factor for myocarditis in those with CD4 counts below 400/cubic cm.

Treatment is mostly supportive +/- treatment of the underlying cause, e.g. steroids for an autoimmune cause. In cases where there are signs of heart failure, drug treatment with aldosterone antagonists, ACEi, beta blockers, diuretics, vasodilators/ nitrates, and potentially anticoagulation for stroke prevention is indicated. If unstable, the patient may require ITU with inotropes, vasopressors, LVAD, or intra-aortic balloon pump.

Pulmonary embolism is a good differential for this presentation, but would not cause the CXR signs or orthopnoea. Dilated cardiomyopathy is also a reasonable explanation for the symptoms, but doesn’t fit the risk factors as well.

39
Q

A 35 year old woman presents to A&E with palpitations, chest pain, and breathlessness. She was on a night out with some friends and admits to having taken a significant amount of cocaine. Her ECG shows a regular narrow complex tachycardia with no visible p waves and a rate of 186bpm. Her blood pressure is 130/87, carotid massage has already been tried to no effect.

What is the most appropriate next step?

A. Start 5mg Bisoprolol OD
B. Immediate DC cardioversion
C. Give 6mg intravenous adenosine
D. Give 50mg intravenous flecainide
E. Give 5mg/kg intravenous amiodarone infusion
A

C. Give 6mg intravenous adenosine

This is a presentation of a supraventricular tachycardia (SVT). SVT is actually a group of different aberrant rhythms, all of which origniate in the atrium or AV node. In practice it is not realistic to differentiate between them on presentation, and even fast atrial fibrillation can be indistinguishable from an SVT. Therefore the use of adenosine in the SVT protocol (after vagal manouevres) is diagnostic as well as therapeutic: adenosine temporarily blocks the AV node and so will reveal the underlying atrial rhythm as well as terminating the tachycardia.

If the initial 6mg bolus is unsuccessful, a 12mg bolus can be given, which can be repeated once more (total 30mg dose of adenosine). After that Verapamil is generally used (though Diltiazem or a beta blocker are also options). If these are unsuccessful then DC cardioversion is indicated.

NB: If the patient becomes haemodynamically unstable, DC cardiovert them

40
Q

Give 5 causes of pericarditis

A

Viral infection or TB (particularly associated with constrictive pericarditis)
Uraemic
Malignant spread or metastasis
Post-MI (early or late)
Post-cardiac surgery
Autoimmune disorders (SLE, rheumatoid arthritis)

NB: any cause of pericarditis can cause constrictive pericarditis (especially TB). This is just the progression of the disease to the point where it causes symptoms of right heart failure.

41
Q

Briefly describe the approach to managing atrial fibrillation

A

A-E approach + history and ECG
Assess haemodynamic stability - cardiovert if unstable
Ascertain duration of symptoms - more than 48 hours requires 3 weeks of anticoagulation first, and rate control during this time
Assess whether to rate or rhythm control
If presenting in fast AF, control rate with beta blocker or rate-limiting CCB (consider amiodarone or digoxin if in heart failure)

For rate control give a beta blocker or rate-limiting CCB 1st line
Digoxin monotherapy may be considered in very sedentary patients
For acute rhythm control - DC cardiovert if present for >48 hours, flecainide otherwise assuming no structural heart damage, amiodarone if there is structural damage

Assess need for long-term rhythm control:
1st line beta blockers
2nd line Dronedarone
3rd line Amiodarone
If drug treatment fails or is unsuitable, left atrial ablation or a ‘pace and ablate’ strategy may be considered

42
Q

A 52 year old woman with a known history of paroxysmal atrial fibrillation which is usually triggered by alcohol and caffeine. The episodes are infrequent and cause mild palpitations. She previously tried taking Bisoprolol but it made her feel dizzy. She is otherwise well with no other significant medical history

How should this woman’s AF be managed long-term?

A. Prescribe Apixaban 5mg BDS
B. Give her an oral 300mg Flecainide dose to take when symptomatic
C. Offer left atrial appendage occlusion
D. Prescribe oral Amiodarone
E. Refer her for left atrial ablation and pacing

A

B. Give her a 300mg Flecainide dose to take when symptomatic

This describes the ‘pill in the pocket’ strategy. This woman is a good candidate because her atrial fibrillation is paroxysmal and seldom affects her, and is mild when it does. She has not tolerated beta blockers, and given how mild her PAF is, it doesn’t seem worth treating her intensively. Instead giving her a dose of Flecainide to take when symptomatic is a safe form of symptom relief. The other options are a bit unwarranted given how mild her condition is.

43
Q

Give causes of general valvular disease:

A
Can cause most:
Infective endocarditis
Rheumatic heart disease
Ischaemic damage
Connective tissue disease
Congenital malformation

Aortic regurgitation:
Aortic dissection
Spondyloarthropithies/ arthritides (ankylosing spondylitis, rheumatoid arhritis)

Aortic stenosis:
Bicuspid aortic valve
Age-related calcification

Mitral stenosis:
Rheumatic heart disease

44
Q

A 35 year old presents to A&E with right-sided pleuritic chest pain. His ECG shows tall broad t-waves with a high take-off and a visible J wave in V2-V5.

What is the most likely cause of the ECG findings?

A. Pericarditis
B. Pulmonary embolism
C. Acute coronary syndrome
D. Early repolarisation
E. Hyperkalaemia
A

D. Early repolarisation

These are changes consistent with benign early repolarisation (BER) - a common finding in young healthy people found in ~10% of people. Given the presentation of this patient to A&E, these findings are not related and are just part of a normal variant. The worrying thing is that BER can mimic ACS or hyperkalaemia, so it is important to take into account the clinical picture and escalate to a senior with a low threshold of suspicion for more serious pathologies. Repeating the ECG 30 minutes later would be a helpful step to differentiate between BER and some more concerning diagnoses.

45
Q

A 73 year old man presents with sudden onset tearing chest pain radiating through to his back. He has a history of poorly controlled hypertension and has been smoking 20 cigarettes per day for 50 years. His ECG is normal and he is haemodynamically stable, but an echocardiogram reveals an intimal tear in the aorta. This is confirmed by a CT which reveals that the lesion extends from the mid-ascending aorta to 2cm distal to the left subclavian artery. An arterial line is inserted and Labetalol is given.

What is the most likely diagnosis, and how should this condition generally be managed?

A. Type A aortic dissection - open repair
B. Type B aortic dissection - open repair
C. Type A aortic dissection - medical mangement +/- endovascular stent
D. Type B aortic dissection - medical mangement +/- endovascular stent

A

A. Type A aortic dissection - open repair

The Stanford classification of aortic dissections is very simple: if it involves any part of the ascending aorta then it is type ‘A’, if not then it is type ‘B’. The landmark here is the left subclavian artery - if the tear begins distal to that, it is type ‘B’. In either type, they should be managed initially with Labetalol +/- GTN to control ther blood pressure, which should be monitored using an arterial line.

Type A needs to be operated on: without surgery 50% of patients die in 48 hours, and 90% will die within a month.

However if they are stable, type B can be initially managed medically by controlling their blood pressure and pain. Complications (haematoma, malperfusion of distal organs or branches, uncontrollable pain, rapid aortic expansion) are indications for endovascular repair.

RCEM summary of aortic dissection:
https://www.rcemlearning.co.uk/reference/aortic-dissection/#1568112704060-230d2cb2-b919

ESC guidelines (p2889):
https://watermark.silverchair.com/ehu281.pdf?token=AQECAHi208BE49Ooan9kkhW_Ercy7Dm3ZL_9Cf3qfKAc485ysgAAAs8wggLLBgkqhkiG9w0BBwagggK8MIICuAIBADCCArEGCSqGSIb3DQEHATAeBglghkgBZQMEAS4wEQQMFCBRp_vpwXDont7cAgEQgIICgi4l6VvcIT8uvmgqgUOhE7r1cfoFAz8Fck1aypVR-Erg0UyGnm-941vjJo_mWb_hr_kqAgxeNvJ4oCBA_sx07UVTKR7RDJN4pZTXOSb7adQooLm7ywZ-2X38W6LiPAhxUlAwlq1IltvA2-mzvKEhklL8CDbi2Qqg-hfxhg0YOeuu_w6dfu_Oxy7Y9l3O9W6k1b45Q0Bf5HjVALiynBl16PuyV4JaoolbAQhUOIRvnwyOgCq-DdkWXmIoOw8tgUW6gSySozNu_w3wApp885ZKgEQ8FPy59PIGoOBJTVf1tzjz7P3mptIiYI9Jro-hUd4-MZI1BR6j6gZaCZ2GhyWg_bElzpqEY-fJRI4scF04PLoxq-ulzfFourw_L_ZinCPIKLzl2XGp_AiKYx742rg_KA5UPRHrv4xYFWCA-iO4SuWehlc7NFNn6gzU7eTiHgYZTxfjcFTbdgvaEpIs9Nkze1ooa6C-yWwgKvoC9cmbg5xOUttyiTP_pHBBH4l8SGrl97EmxLR3ju8C7uX-CMdZ-2uPYs4inLnRsxBIynoJKDj8OUqDPZpJUmptkK54T2tc41xmSVa2LXD8xdyk7iBCeFyCp2OPOzAvK8Rw3MNeAM6ATa7vc1ulF-gNfJcAYgKApNdmvnTNmckDulOKx8PTpnTwQrNihvzotF0MzKCs2LmDsVmwr0WNMVhmY3doJaliI8cgicIz6_f5JX0zH6FTfok2y34EEVjHjnGqgVeniQJzh1RZ8Q4RHTpwsl9Nau0p048AyVgoPqebQTmkLUy6xPpO9UqJ3Z9d2DsClqQe7hbCbiq3UuzMXS8HJTmc3Uk5vJeSLXpVFPxkciH1q2nSUY9IFQ
46
Q

A 24 year old woman presents to A&E after an episode of palpitations and breathlessness. This has happened a few times in the past 2 months and she is starting to get worried. She is otherwise fit and well. There are no medical conditions in her family, but she mentions her father died of a heart attack when she was 8. On examination she has an irregularly irregular pulse though it resolves spontaneously. Her observations are all normal, and an ECG shows deep inverted T waves and large QRS complexes in V2-V5 and prominent Q waves in V5, V6, II, III, and aVF.

What is the most likely diagnosis?

A. Dilated cardiomyopathy
B. Previous STEMI
C. Current NSTEMI
D. Wolf-Parkinson-White syndrome
E. Hypertrophic obstructive cardiomyopathy
A

E. Hypertrophic obstructive cardiomyopathy

HOCM is the most common inherited cardiomyopathy, affecting ~1 in 500 people (estimates vary significantly by study/ population). It is an autosomal dominant condition which causes inwards hypertrophy of cardiac muscle leading to valve function impairment, diastolic dysfunction/ impaired coronary perfusion, and left outflow obstruction. Because of its hereditary nature, the family of someone diagnosed with HOCM should be screened using ECG and echo.

It may present as angina and dyspnoea as these are direct consequences of outflow obstruction and impaired coronary artery filling, or it may present with palpitations. This is because HOCM is associated with arrythmias, most commonly atrial fibrillation.

Symptomatic relief is with beta blockers, verapamil, or disopyramide. If the patient has also developed atrial fibrillation, they should be anticoagulated as normal. In those with risk factors for sudden cardiac death, and implantable carciac defibrillator is offered. Surgical removal of the hypertrophied muscle (septal myotomy-myectomy) may be offered for those who don’t tolerate medical treatment, or are symptomatic despite it.

RCP HOCM guidelines:
https://www.rcpjournals.org/content/clinmedicine/7/4/383.full.pdf

ESC HOCM guidelines:
https://academic.oup.com/eurheartj/article/35/39/2733/853385

47
Q

A 28 year old woman presents to her GP with 6 weeks of steadily progressive breathlessness and reduced exercise tolerance. The breathlessness is worst when she lies down at night, and she has noticed her ankles have started to swell. She was treated for ALL as a child with chemotherapy. She has no recent travel history. On examination, she has bilateral fine crackles, a pan systolic murmur heard best in the midclavicular line in the 5th intercostal space, and pitting oedema to the mid-shin.

What would be the most useful investigation?

A. Serum BNP
B. Serum thiamine levels
C. Echocardiogram
D. Blood film + PCR
E. ECG
A

C. Echocardiogram

This is a history of dilated cardiomyopathy (DCM) - one of the most common forms of cardiomyopathy. DCM features eccentric enlargement of the ventricles i.e. more sarcomeres are added side by side rather than enlarging inwards as in HOCM. An echocardiogram is the most useful investigation here because not only can it assess ejection fraction, it can be used to visualise the heart structure and analyse its movements, thereby diagnosing the cause of the heart failure. BNP would diagnose the heart failure, but not the cause of it.

DCM is most commonly idiopathic, but other well established causes include infection, genetic syndromes/ inherited predisposition, alcoholism, chemotherapy agents (especially doxorubicin), thiamine deficiency, and pregnancy. The risk factor in this woman’s history is her history of childhood cancer and treatment with chemotherapy.

The most common causative infection worldwide is trypanasoma cruzei (Chagas disease), though TB and Coxsackie B virus are also established causes. Pregnancy may cause DCM in the peri-partum period, but 50% of cases will spontaneously resolve.

Treatment of DCM is essentially treatment of heart failure (ACEi, beta blockers, aldosterone antagonists) +/- LVAD and heart transplant.

48
Q

A 52 year old woman is brought to A&E by ambulance having been found collapsed on the street. She is unconscious so the paramedic is maintaining her airway, she is saturating at 91% on 15L, and her heart rate is 32bpm with a BP of 72/49. Her ECG shows Mobitz II heart block.

What is the best next step?

A. Urgent transfer to cathlab
B. Give 120mg intravenous flecainide
C. Immediate DC cardioversion
D. Immediate thrombolysis
E. Give 500mcg intravenous atropine
A

E. Give 500mcg intravenous atropine

The ALS algorithm for bradycardia is simpler than the one for tachycardia: if there is syncope, or features of shock, heart failure, or myocardial ischaemia then give atropine. If there are none of these features but there is a risk of asystole (recent asystole, Mobitz II heart block, 3rd degree heart block etc.) then give atropine. Other options include adrenaline, isoprenaline, and transcutenaous pacing, but atropine is the best first intervention to increase heart rate and cardiac output.

Unlike in tachyarrythmias, cardioversion is not immediately indicated when the patient becomes shocked.

ALS guidelines (see 6):
https://www.resus.org.uk/library/2015-resuscitation-guidelines/peri-arrest-arrhythmias
49
Q

Name some indications for an ICD:

A

Primary prevention:

Inherited cardiac conditions predisposing to ventricular arrthymia (Brugada, HOCM, long QT syndrome)
Previous MI patients with LVEF <35% despit optimal medical treatment
Some types of previous cardiac surgery

Secondary prevention:

Previous ventricular arrhythmia with a non-reversible cause causing arrest, haemodynamic compromise or heart failure

50
Q

A 24 year old man is found collapsed on the street and an ambulance is called. He is in cardiac arrest and the paramedics start CPR. The AED detects ventricular fibrillation and he is successfully shocked back into sinus rhythm. Later in the hospital his ECG shows ST elevation with T-wave inversion in V1-V3. During the history he reveals his dad passed away from a heart attack at the age of 37.

What is the most likely diagnosis?

A. Long QT syndrome
B. Brugada syndrome
C. STEMI
D. Hypertrophic obstructive cardiomyopathy
E. Dilated cardiomyopathy
A

B. Brugada syndrome

Brugada syndrom is a genetic mutation in cardiac sodium channels that predisposes to dangerous ventricular arrhythmias. It can be inherited via an autosomal dominant mechanism, but ~50% of cases are due to de novo mutations. It is most common in Southeast Asia/ people of Southeast Asian descent. The ony definitive treatment is implantation of an implantable cardiac defibrillator to terminate ventricular arrhyhmias. This is important as otherwise the yearly mortality of Brugada syndrome is 10%.

The ECG finding described is Brugada sign: ST elevation in >1 lead in V1-V3 with T wave inversion following it. Though not diangostic on its own, in combination with the ventricular fibrillation and family history of early crdiac death, it point towards Brugada syndrome.

51
Q

A 58 year old man presents with severe, sudden onset, sharp chest pain and arrives in A&E by ambulance 3 hours after symptom onset. He has hypertension and has smoked 30 a day for 30 years. On examination there is a mid diastolic murmur heard best in the left 3rd intercostal space parasternally. His radial pulses are unequal and his BP is 163/118 on the right and 142/87 on the left. An ECG shows ST depression in leads II, III, and aVF. His 4 hour troponins are not elevated.

How should this patient be managed?

A. Immediate transfer to theatres
B. Give aspirin, prasugrel, and morphine, and send for immediate PCI
C. Give aspirin, ticagrelor, fondaparinux, and morphine
D. Start apixaban and send for an urgent CTPA
E. Give I.V. labetalol and morphine, insert an arterial line, and order an urgent CT

A

E. Give I.V. labetalol and morphine, insert an arterial line, and order an urgent CT

This is a history of aortic dissection: a tear in the intimal layer of the artery which allows blood to be forced in and dissect the intima away from the arterial wall. The key features in this history are severe sudden onset chest/back pain, hypertension, smoking, and unequal arm pulses and pressures. ST depression may be seen on an ECG, but cardiac enzymes are usually not significantly elevated. A chext x-ray may show a widened mediastinum, an enlarged heart due to haemopericardium, or haemothorax.

Aortic dissection can present with stroke symptoms due to occlusion of the carotid arteries. The dissection may also extend to disrupt the aortic valve and cause regurgitation (as has happened here). Dissections may rarely cause inferior ischaemic changes incluidng ST elevation, as the right coronary artery can be occluded by the dissection.

Significant other risk factors include connective tissue disease (Ehlers-Danlos, Marfan’s), aortic aneurysm, family history of dissection, aortic coarctation, and bicuspid aortic valve.

Aortic dissections are classified using the Stanford classification:
If the ascending aorta is involved (proximal to the left subclavian artery) it is type A
Anything else is type B
Type A dissections require surgical repair given the very high mortality. However type B dissections can be managed medically with blood pressure control unless complications arise e.g. intractable pain, end-organ malperfusion, rapid expansion, haematoma.

52
Q

A 40 year old woman presents to A&E with palpitations and breathlessness. Her HR is 140bpm, SO2 is 96% RA, BP is 110/85, and her ECG shows a broad complex regular tachycardia.

What is the best next step in controlling this arrythmia?

A. Adenosine 6mg I.V. rapid bolus
B. Amiodarone 300mg I.V. over 30-60 minutes
C. Adenosine 12mg I.V. rapid bolus
D. Start a beta blocker
E. DC cardioversion
A

B. Amiodarone 300mg I.V. over 30-60 minutes

The Resus Council have produced a very handy flow diagram on peri-arrest arrythmias (linked below). The important questions to ask (in order) are:
Is the patient haemodynamically stable?
Is the QRS narrow or broad?
Is the rhythm regular or irregular?

Based on these questions, the algorithm leads you to a probable diagnosis and initial management. The probable diagnosis here is ventricular tachycardia, so the appropriate treatment is amiodarone 300mg I.V. over 30-60 minutes, followed by a 900mg infusion over 24 hours.

ALS peri-arrest arrhythmias guideline:
https://www.resus.org.uk/library/2015-resuscitation-guidelines/peri-arrest-arrhythmias

53
Q

ALS

A

REVIEW*

54
Q

A 65 year old man presents to A&E with crushing central chest pain. The clerking doctor is unsure whether he has unstable angina, or is having a myocardial infarction.

Which of the following is most useful for distinguishing the two?

A. BNP
B. D-dimer
C. Troponins
D. ECG
E. Echocardiogram
A

C. Troponins

Unstable angina and NSTEMI are very difficult to differentiate. They present similarly and have similar ECG findings, but troponins will be elevated in an NSTEMi, but not generally in unstable angina.

55
Q

Driving rules for MI

A

Don’t have to inform the DVLA but can’t drive for:

1 week if you had angioplasty, it was successful and you don’t need any more surgery
4 weeks if you had angioplasty after a heart attack but it wasn’t successful
4 weeks if you had a heart attack but didn’t have angioplasty

If you are a bus, coach, or lorry driver you must stop driving for 6 weeks, inform the DVLA, and then have a GP +/- DVLA assessment

56
Q

Summarise the types of pacemaker:

A

Righ atrium only: SA node disease in young people with completely healthy AV node
Right ventricle only: for pacing people in permament atrial fibrillation
Right atrium and ventricle: everything else

57
Q

A 68 year old man is seen by his GP with a 3 month history of worsening dyspnoea. He also admits to central chest pain triggered by exertion and relieved by rest for the past year. He has not lost conciousness and his activities of daily living are not affected. He has otherwise been well, has no medical conditions bar hypertension managed with lisinopril, and a standard set of bloods and obs are clear. The GP refers him for an echocardiogram which reveals a stenosed aortic valve.

How should this patient be managed?

A. Give GTN spray reliever + antiplatelet prophylaxis
B. TAVR
C. Open valve replacement
D. Start beta blockers
E. Balloon valvuloplasty
A

C. Open valve replacement

Aortic stenosis is narrowing and stiffening of the aortic valve; in young patients it is usually due to a bicuspid valve, and in older patients is most often due to calcification. Once it becomes symptomatic the average life expectancy is 2-3 years, though open surgical repair can return them to a near-normal life expectancy. Transcatheter aortic valve replacement is an option in patients too unwell to undergo open surgery; it has a lower long-term survival rate but this influenced by the cohort of people it is given to.

This patient is relatively well and a suitable candidate for open repair. Balloon valvuloplasty is a temporary bridge to treatment used to relieve stenosis of the valve.